¿Por qué se necesitan anticonmutadores en la cuantización de los campos de Dirac?

¿Por qué es realmente necesario el anticonmutador en la cuantización canónica del campo libre de Dirac?

Hay una discusión sobre esta pregunta en Peskin y Schroeder, An Introduction to QFT, sección 3.5.
Respuesta corta: de esta manera las partículas de Dirac siguen la regla de exclusión de Pauli-Fermi

Respuestas (2)

La razón más elemental es que el hamiltoniano del campo de Dirac está acotado por debajo solo cuando usa relaciones anticonmutación en los operadores de creación/aniquilación en lugar de conmutadores. Una teoría del campo cuántico libre con energía ilimitada por debajo no tiene un vacío estable.

Es más fácil demostrar esto en dos dimensiones, donde no hay problemas de polarización.

Ejemplo instructivo 2d

En dos dimensiones (un espacio, un tiempo), hay un buen análogo dimensionalmente reducido, que es el Majorana-Weyl Fermion que se mueve hacia la derecha (necesariamente sin masa) (el argumento también funciona con fermiones de Dirac 2d con dos componentes, pero este es el más simple caso). Este es un campo de un solo componente ψ que obedece a la ecuación de movimiento

( t X ) ψ = 0

Esta ecuación simple se deriva de la ecuación de Dirac 2d usando las matrices de Dirac 2d (0,1;-1,0) y (0,1;1,0) (convención real, explícitamente real), que son γ 0 = σ X y γ 1 = i σ y . Se elevan al cuadrado a 1 y -1 respectivamente, y son anticonmutadores, por lo que reproducen el tensor métrico dimensional 1+1. los γ 5 analógico, al que llamaré Γ para acomodar diferentes dimensiones, es diagonal en esta representación explícita, y Γ = σ z .

Los dos vectores propios de Γ propagarse independientemente por la ecuación de movimiento sin masa 2d

γ i i ψ = 0

Y además, porque el γ matrices son reales, esta es una representación de Majorana (la mayoría de los físicos escriben la ecuación de dirac con un factor i delante de la derivada, de modo que las matrices de Dirac para una representación de Majorana son puramente imaginarias. Estoy usando una convención matemática para esto, porque Me gusta que las ecuaciones de movimiento sean reales. A otros les gusta que el propagador del espacio k no tenga factores de i en la parte k. Desafortunadamente, los físicos nunca se establecieron en una convención sensata única: todos tienen su propia forma preferida de escribir Dirac matrices). Entonces es sensato en la ecuación de movimiento restringir ψ ser hermitiano, ya que su conjugado hermitiano obedece exactamente a la misma ecuación.

Para que el campo tenga una descomposición k

ψ ( X ) = a k mi i k X i k t d k

Y la condición de realidad (Hermiticidad) te dice que a ( k ) = a ( k ) (habría que decir que la normalización de la a La expansión de los operadores no es completamente trivial desde el punto de vista conceptual --- la a están normalizados relativista y no relativistamente, porque la polarización del espinor w cancela el factor de hipérbola de capa de masa, de modo que la integración dk no está ponderada por nada, es solo el cálculo normal integral con medida uniforme)

Un operador con frecuencia definida, que (imagen de Heisenberg) evoluciona en el tiempo de acuerdo con

t O = i ω O

Tiene la propiedad de que es un operador ascendente --- al actuar con este operador se suma ω a la energía. Si ω es negativo, a es un operador de aniquilación. La condición de que el vacío sea estable dice que todos los operadores de aniquilación dan 0 cuando actúan sobre el estado de vacío.

Pero observe que la frecuencia en la expansión de ψ cambia de signo en k = 0 . Esto provino de la linealidad del hamiltoniano de Dirac en los momentos. Significa que el operador a k actúa para aumentar la energía para k>0, pero actúa para disminuir la energía para k < 0 . Esto significa que el k > 0 los operadores crean, y el k < 0 operadores aniquilar, por lo que la forma correcta de a ( k ) son operadores de creación, mientras que los k < 0 Los operadores son operadores de aniquilación.

El operador de energía cuenta el número de partículas de momento k y multiplica por su energía:

H = k > 0 k a ( k ) a ( k ) d k

Y este manifiestamente no es un operador local, se define solo integrado sobre k>0. Para convertirlo en un operador local, debe extender la integración a todos los k, pero luego las contribuciones de k negativas y k positivas tienen signos opuestos y deben ser iguales. Para arreglar esto, debe tomar relaciones de anticonmutación

{ a ( k ) , a ( k ) } = i d ( k k )

Y entonces

H = 1 2 k a ( k ) a ( k ) = ψ ( X ) i X ψ ( X ) d X

Tenga en cuenta que esto parece ser un derivado perfecto, y lo sería si ψ no eran cantidad anticonmutación. Para cantidades anticonmutación,

X ψ 2 = ψ X ψ + X ψ ψ

Que es cero, debido a la anticonmutación.

Razones más profundas

Aunque esto parece una propiedad accidental, que la energía fuera negativa sin anticonmutadores, no lo es. La razón más profunda se explica con la teoría del campo euclidiano usando un formalismo de Feynman-Schwinger, pero esto requiere la comprensión de las versiones euclidiana e integral de camino de los campos anticonmutantes, lo que requiere sentirse cómodo con las cantidades anticonmutantes, lo que requiere una motivación. Así que es mejor aprender primero la razón superficial.

¿Por qué necesito que todas las relaciones sean anti-conmutación? Puedo tomar 2 de ellos como anti-conmutación, pero el tercero, es decir, la relación entre el operador de creación y aniquilación, es conmutativo y aún mantiene la exclusión de Pauli. física.stackexchange.com/q77384

Suponiendo que la pregunta es por qué anticonmutadores en lugar de conmutadores, según el comentario de David:

Si denoto las propiedades de las partículas de Dirac por un índice general α - estos pueden incluir giros, momentos - entonces si creo un estado de dos partículas, con la partícula 1 teniendo α 1 y partícula 2 α 2 entonces el estado se da aplicando operadores de creación en el orden

| α 1 α 2 = b α 1 b α 2 | 0 >
Si los creamos al revés:
| α 2 α 1 = b α 2 b α 1 | 0 >
entonces si las b obedecen a relaciones de anticonmutación , es fácil ver que
| α 2 α 1 = | α 1 α 2
es decir, la teoría reproduce de forma natural las estadísticas de Fermi, como se desearía para las partículas de espín 1/2 de Dirac.

Sí, esto es cierto, pero creo que el OP quería una prueba de estadísticas de giro. Di la prueba en el caso más simple, 2d majorana Weyl spinors.
Sí, no estaba seguro de a qué nivel estaba dirigida la pregunta: opté por "por qué anticonmutadores en lugar de conmutadores". Respondiste la pregunta más difícil (+1 para una prueba simple)
Vale, +1 para ti también.